Pytanie:
Czy dH (S, P) = dH (T, P) prawda (zawsze)?
khaverim
2017-10-13 18:44:15 UTC
view on stackexchange narkive permalink

Dzisiaj trafiłem na interesujące pytanie z kilkoma studentami chemii fizycznej. Na podstawie poniższych kroków nie jesteśmy pewni, czy stwierdzenie w tytule jest / może być prawdziwe. Zakładając, że $ dN = 0 $,

Naturalnymi zmiennymi entalpii są Entropia i Ciśnienie:

$$ dH (S, p) = TdS + Vdp \ space (1) $$

Entalpię można wyrazić jako całkowitą pochodną temperatury i ciśnienia:

$$ dH (T, p) = \ frac {\ częściowe H} {\ częściowe T} \ vert_p \ space dT + \ frac {\ części H} {\ części p} \ vert_T \ space dp \ space (2) $$

Całkowita entalpia wynosi $$ H = U + pV = TS - pV + \ sum_i \ mu_i N_i + pV = TS + \ sum_i \ mu_i N_i \ space (3) $$

Zatem biorąc pochodne cząstkowe w (2), $$ dH (T, p) = SdT + 0 \ space dp \ space (4) $$

Oznacza to, że jeśli $ dH (S, p) = dH (T, p) $,

$$ SdT = TdS + Vdp \ space (5) $$ musi mieć wartość true. Czy ktoś może zobaczyć coś nie tak?

Wydaje się, że sprowadza się to do pytania matematycznego: czy dwie różniczki tej samej funkcji (zawsze) są równe, jeśli są wyrażone przez różne zmienne?

Nie zgadzam się z twoim stwierdzeniem w $ (3) $, że $ U = TS-pV $. Skąd to pochodzi?
https://en.wikipedia.org/wiki/Internal_energy#Internal_energy_of_multi-component_systems - może to złe założenie, ponieważ T i P są zmienne?
Ta linia $ U = TS-pV + \ sum_i \ mu_i N_i $ faktycznie mówi, że $ G = \ sum_i \ mu_i N_i = U-TS + pV $ gdzie $ G $ to energia swobodna Gibbsa. Zaniedbałeś chemiczny potencjał $ \ mu $ różnych gatunków w twoim systemie - który będzie miał ukryty i złożony związek z $ T $ i $ p $.
To prawda, dzięki. Niemniej jednak moje pytanie pozostaje w centrum uwagi. Częściowe pochodne H są nadal takie same (brak zależności $ \ mu $ lub $ N $)
https://en.wikipedia.org/wiki/Enthalpy#Other_expressions prawdopodobnie zapewnia zależność, której potrzebujesz dla dokładnej różnicy, jeśli $ H (T, p) $ z odniesieniem do wyprowadzenia - ale z reguły $ \ frac {\ częściowe \ mu} {\ częściowe T} \ ne 0 $, stąd czwarte równanie jest nieprawidłowe.
W odniesieniu do potencjału chemicznego odpowiedź na https://physics.stackexchange.com/questions/168704/chemical-potential-as-a-function-of-temperature również jest dość pouczająca.
Wyobrażam sobie, że problem polega na tym, że $ (\ częściowe H / \ częściowe T) | _P = S $, ponieważ, jak wspomniał użytkownik213305, inne zmienne termodynamiczne również będą zależeć od $ T $. Z drugiej strony odpowiedź na twoje tytułowe pytanie brzmi: tak, ponieważ w tym przypadku używamy po prostu różnych zmiennych do opisania tego samego.
Widzę. Zależność ciśnienia od temperatury w $ \ mu $ powinna prowadzić do innego wyniku w (4). Nie wiem, jaki to wynik, ale jeśli ktoś może udowodnić, że to $ TdS + Vdp $, to będę zadowolony, że $ dH = dH $ niezależnie od zmiennych
Jeden odpowiedź:
Ignacio
2017-10-15 11:12:56 UTC
view on stackexchange narkive permalink

Podane rozumowanie jest częściowo poprawne, ale ostateczna relacja, do której doszedłeś, jest nieprawidłowa. Spróbuję wyjaśnić dlaczego i wyraźnie napiszę zależność N dla kompletności. Najważniejsze jest to, że kiedy ktoś pisze wyrażenie takie jak $$ \ left (\ frac {\ częściowe H} {\ częściowe T} \ right) _ {P, N} $$ to tak naprawdę ma na myśli "wziąć pochodną cząstkową $ H $ zapisane jako funkcja $ T $, $ P $ i $ N $ w odniesieniu do $ T $ ". Kiedy weźmiesz częściowe pochodne w równaniu (2), powinieneś wziąć je pod uwagę, biorąc pod uwagę $ H $ jako funkcję $ T $, $ P $ i $ N $. Musisz więc rozważyć wyrażenie $ H = H (T, P, N) = S (T, P, N) T + \ mu (T, P) N $. Jeśli rozróżnisz to równanie w odniesieniu do T i P, otrzymamy:

$$ \ left (\ frac {\ częściowe H} {\ częściowe P} \ right) _ {T, N} = \ lewo (\ frac {\ częściowe S} {\ częściowe P} \ prawe) _ {T, N} T + \ left (\ frac {\ części \ mu} {\ częściowe P} \ right) _ {T} N ~ ~~; ~~~ \ left (\ frac {\ częściowe H} {\ częściowe T} \ right) _ {P, N} = \ left (\ frac {\ częściowe S} {\ częściowe T} \ right) _ {P, N} T + S + \ left (\ frac {\ części \ mu} {\ częściowe T} \ right) _ {P} N $$ Jeśli zastąpisz te dwie relacje w swoim równaniu (2), wynikiem jest :

$$ \ mathrm {d} H = \ left (\ left (\ frac {\ częściowe S} {\ częściowe P} \ right) _ {T, N} T + \ left (\ frac {\ części \ mu} {\ częściowe P} \ prawej) _ {T} N \ right) \ mathrm {d} P + \; \ left (\ left (\ frac {\ częściowe S} {\ częściowe T} \ po prawej) _ {P, N} T + S + \ left (\ frac {\ części \ mu} {\ częściowe T} \ right) _ {P} N \ right) ~ \ mathrm {d} T. $$

Rzeczywiście możesz to zrównać ze swoim równaniem (1), które zadajesz w swoim głównym pytaniu. To jest to samo, co zwykle robi się, wyrażając skalar jako funkcję różnych zestawów współrzędnych, na przykład $ f = f (x, y) = x ^ 2 + y ^ 2 $ i $ f = f (r, \ theta) = r ^ 2 $ oznacza to, zrównując oba, że ​​$ x ^ 2 + y ^ 2 = r ^ 2 $, czyli relacja, która musi być zachowana, jeśli chcesz zarówno $ (x, y) $, jak i $ (r , \ theta) $ w odniesieniu do $ f $ (to ostatnie zdanie może być nieco tautologiczne, mam nadzieję, że to, co to oznacza, jest jasne, zauważ, że z pewnością nie jest „zawsze” prawdą, że $ f (x, y) $ i $ f (r, \ theta) $ są równe, ponieważ są to dwie różne funkcje, aczkolwiek wyrażone tą samą literą, na przykład $ f (x = 1, y = 0) = 1 $ ale $ f (r = 2, \ theta = \ pi) = 4 $, musi istnieć określona zależność między współrzędnymi, aby były one równe). Jeśli to zrobisz, otrzymasz:

$$ \ left (\ left (\ frac {\ częściowe S} {\ częściowe P} \ right) _ {T, N} T + \ left (\ frac { \ części \ mu} {\ częściowe P} \ prawej) _ {T} N \ prawej) \ mathrm {d} P + \; \ left (\ left (\ frac {\ częściowe S} {\ częściowe T} \ right ) _ {P, N} T + S + \ left (\ frac {\ części \ mu} {\ częściowe T} \ right) _ {P} N \ right) ~ \ mathrm {d} T = T ~ \ mathrm { d} S + V ~ \ mathrm {d} P. $$

Zwróć uwagę, że otrzymasz to samo wyrażenie, nawet jeśli weźmiesz pod uwagę procesy, w których $ \ mu \ mathrm {d} N $ nie było cero, ponieważ oba warunki zostałyby anulowane. Jeśli ktoś pamięta, że ​​$ T \ mathrm {d} S = T \ left (\ frac {\ częściowe S} {\ częściowe P} \ right) _ {T, N} \ mathrm {d} P + T \ left (\ frac {\ części S} {\ częściowe T} \ right) _ {P, N} \ mathrm {d} T $ to upraszcza to do:

$$ \ left (\ left (\ frac { \ częściowe \ mu} {\ częściowe P} \ prawej) _ {T} N \ prawej) \ mathrm {d} P + \; \ left (S + \ left (\ frac {\ part \ mu} {\ częściowe T} \ right) _ {P} N \ right) ~ \ mathrm {d} T = V ~ \ mathrm {d} P. $$

Dzielenie przez N:

$ $ \ left (\ frac {\ part \ mu} {\ part P} \ right) _ {T} \ mathrm {d} P + \; \ left (\ bar {S} + \ left (\ frac {\ part \ mu} {\ częściowe T} \ right) _ {P} \ right) ~ \ mathrm {d} T = \ bar {V} ~ \ mathrm {d} P. $$

To jest prawdziwe wtedy i tylko wtedy, gdy:

$$ \ left (\ frac {\ części \ mu} {\ częściowe P} \ right) _ {T} = \ bar {V} ~~; ~~ \ left (\ frac {\ części \ mu} {\ Partial T} \ right) _ {P} = - \ bar {S}. $$

Są to poprawne relacje, które można również wywnioskować z równania Gibbsa-Duhema: $ N \ mathrm { d} \ mu -V \ mathrm {d} P + S \ mathrm {d} T = 0 $.



To pytanie i odpowiedź zostało automatycznie przetłumaczone z języka angielskiego.Oryginalna treść jest dostępna na stackexchange, za co dziękujemy za licencję cc by-sa 3.0, w ramach której jest rozpowszechniana.
Loading...